How is going around the circle once in each direction homotopic to a point?

The name of the pictureThe name of the pictureThe name of the pictureClash Royale CLAN TAG#URR8PPP












9












$begingroup$


Two paths are homotopic if one can be continuously deformed to the other, right? So I've been told that the fundamental group of the circle is isomorphic to the integers, since you can't deform e.g., the path that goes around once clockwise to that which goes around twice clockwise, since the disc gets in the way, so it's generated by the the clockwise and counterclockwise paths. But that seems to imply that if you go around once clockwise and once counterclockwise, you get a path that can be continuously deformed to the base point? How?










share|cite|improve this question









$endgroup$
















    9












    $begingroup$


    Two paths are homotopic if one can be continuously deformed to the other, right? So I've been told that the fundamental group of the circle is isomorphic to the integers, since you can't deform e.g., the path that goes around once clockwise to that which goes around twice clockwise, since the disc gets in the way, so it's generated by the the clockwise and counterclockwise paths. But that seems to imply that if you go around once clockwise and once counterclockwise, you get a path that can be continuously deformed to the base point? How?










    share|cite|improve this question









    $endgroup$














      9












      9








      9





      $begingroup$


      Two paths are homotopic if one can be continuously deformed to the other, right? So I've been told that the fundamental group of the circle is isomorphic to the integers, since you can't deform e.g., the path that goes around once clockwise to that which goes around twice clockwise, since the disc gets in the way, so it's generated by the the clockwise and counterclockwise paths. But that seems to imply that if you go around once clockwise and once counterclockwise, you get a path that can be continuously deformed to the base point? How?










      share|cite|improve this question









      $endgroup$




      Two paths are homotopic if one can be continuously deformed to the other, right? So I've been told that the fundamental group of the circle is isomorphic to the integers, since you can't deform e.g., the path that goes around once clockwise to that which goes around twice clockwise, since the disc gets in the way, so it's generated by the the clockwise and counterclockwise paths. But that seems to imply that if you go around once clockwise and once counterclockwise, you get a path that can be continuously deformed to the base point? How?







      algebraic-topology fundamental-groups






      share|cite|improve this question













      share|cite|improve this question











      share|cite|improve this question




      share|cite|improve this question










      asked Jan 30 at 12:37









      user361424user361424

      1,313414




      1,313414




















          3 Answers
          3






          active

          oldest

          votes


















          13












          $begingroup$

          As @Ben says, the key is that the middle point, where you switch from finishing the first loop to starting the second loop, is able to move. Here's some bad pictures to feed your intuition a bit.



          If you wrap a rubber band once around a post and let go, it will stay around the post:



          enter image description here



          On the other hand, if you wrap it once, then wrap it the other way, you get this (and when you let go, the band will snap back and have nothing to do with the post):



          enter image description here






          share|cite|improve this answer









          $endgroup$




















            9












            $begingroup$

            The reason one clockwise/counterclockwise loop is not nulhomotopic is that both the beginning and end of the loop must stay fixed. If you were allowed to move either one of the endpoints during the homotopy, then this path would be contractible. If you do one clockwise then one counter clockwise loop, then only the start of the first clockwise loop and end of the second counterclockwise loop must remain fixed, while the middle point is allowed to move. This allows you to contract the whole loop.



            Concretely, if
            $$f(t) = begincases e^4pi i t&tleq 1/2\ e^-4pi i t&1/2leq t leq 1 endcases$$



            then there is a nulhomotopy $$h(t,s) = begincasesf(ts)&t leq 1/2\f(s(1-t)) & 1/2leq tendcases$$






            share|cite|improve this answer











            $endgroup$








            • 1




              $begingroup$
              Presumably start and beginning should be beginning and end?
              $endgroup$
              – timtfj
              Jan 30 at 14:39










            • $begingroup$
              @timtfj Fair enough :)
              $endgroup$
              – Ben
              Jan 30 at 15:45










            • $begingroup$
              The $h$ you provide is not nullhomotopic. Indeed, check the endpoints are fixed for all values of $s$. they are correct for $0$ and $1$ but nowhere in between. What you describe is a line that makes it $frac1s$ of the way around the path that is one loop one way, one loop back. The problems are: you wish the condition ($t$ less/greater than $0.5$) to remain true of $t$, not the function argument, and, the function will not be smooth at switch in the current form.
              $endgroup$
              – drjpizzle
              Jan 31 at 11:15











            • $begingroup$
              @drjpizzle Yes I can see it's not right - I think $f(st)$ should be correct for $t leq 1/2$, but not when $t geq 1/2$.
              $endgroup$
              – Ben
              Jan 31 at 11:19










            • $begingroup$
              Well it's smooth and you have a nulhomotopy exists so you can extend it to one... I have put the formula as a comment to my answer (in response to yours.)
              $endgroup$
              – drjpizzle
              Jan 31 at 11:36


















            0












            $begingroup$

            I think this is best answered directly/constructively:



            Going a fraction $k$ about the circle one way and them coming back again, as $k$ varies smoothly between $0$ and $1$ should do the trick.



            At $k = 0$ this is the point. At $k = 1$ this is one around and once back. I think its clear it's continuious.






            share|cite|improve this answer









            $endgroup$












            • $begingroup$
              Why it is right in the general case (the winding number arguments given) is a useful result and nice answers. It's the right way to think about it, and understand why this is a nice result. But to the particular question answered, I think this is the most direct answer.
              $endgroup$
              – drjpizzle
              Jan 30 at 20:30











            • $begingroup$
              By the way this is what the nulhomtopy $h(t,s)$ in a previous answer does, if you plug in $k$ (in this answer) in the $s$ variable.
              $endgroup$
              – Ben
              Jan 31 at 10:44










            • $begingroup$
              @Ben This is not the case. In your example, take $f(st)$ for $s = 0.5$. This becomes: $e^2 pi i t$ or one full loop. Not half a loop forward and half a loop back. I know what you are getting at but I think you made a mistake. I will comment on your answer to see if i can make this clear.
              $endgroup$
              – drjpizzle
              Jan 31 at 11:06











            • $begingroup$
              Thanks you're right. I'll try to fix the formula for $h$, if you have any suggestions please let me know.
              $endgroup$
              – Ben
              Jan 31 at 11:14










            • $begingroup$
              @Ben I think in this case, a wordy answer might be best. But: $h(t,s) = e^2s ((2 pi) t i)$ when $t < 0.5$, $e^2s ((2 pi) (1 - t) i)$ otherwise, I think is correct. edit: grammar and silly mistakes.
              $endgroup$
              – drjpizzle
              Jan 31 at 11:25











            Your Answer





            StackExchange.ifUsing("editor", function ()
            return StackExchange.using("mathjaxEditing", function ()
            StackExchange.MarkdownEditor.creationCallbacks.add(function (editor, postfix)
            StackExchange.mathjaxEditing.prepareWmdForMathJax(editor, postfix, [["$", "$"], ["\\(","\\)"]]);
            );
            );
            , "mathjax-editing");

            StackExchange.ready(function()
            var channelOptions =
            tags: "".split(" "),
            id: "69"
            ;
            initTagRenderer("".split(" "), "".split(" "), channelOptions);

            StackExchange.using("externalEditor", function()
            // Have to fire editor after snippets, if snippets enabled
            if (StackExchange.settings.snippets.snippetsEnabled)
            StackExchange.using("snippets", function()
            createEditor();
            );

            else
            createEditor();

            );

            function createEditor()
            StackExchange.prepareEditor(
            heartbeatType: 'answer',
            autoActivateHeartbeat: false,
            convertImagesToLinks: true,
            noModals: true,
            showLowRepImageUploadWarning: true,
            reputationToPostImages: 10,
            bindNavPrevention: true,
            postfix: "",
            imageUploader:
            brandingHtml: "Powered by u003ca class="icon-imgur-white" href="https://imgur.com/"u003eu003c/au003e",
            contentPolicyHtml: "User contributions licensed under u003ca href="https://creativecommons.org/licenses/by-sa/3.0/"u003ecc by-sa 3.0 with attribution requiredu003c/au003e u003ca href="https://stackoverflow.com/legal/content-policy"u003e(content policy)u003c/au003e",
            allowUrls: true
            ,
            noCode: true, onDemand: true,
            discardSelector: ".discard-answer"
            ,immediatelyShowMarkdownHelp:true
            );



            );













            draft saved

            draft discarded


















            StackExchange.ready(
            function ()
            StackExchange.openid.initPostLogin('.new-post-login', 'https%3a%2f%2fmath.stackexchange.com%2fquestions%2f3093470%2fhow-is-going-around-the-circle-once-in-each-direction-homotopic-to-a-point%23new-answer', 'question_page');

            );

            Post as a guest















            Required, but never shown

























            3 Answers
            3






            active

            oldest

            votes








            3 Answers
            3






            active

            oldest

            votes









            active

            oldest

            votes






            active

            oldest

            votes









            13












            $begingroup$

            As @Ben says, the key is that the middle point, where you switch from finishing the first loop to starting the second loop, is able to move. Here's some bad pictures to feed your intuition a bit.



            If you wrap a rubber band once around a post and let go, it will stay around the post:



            enter image description here



            On the other hand, if you wrap it once, then wrap it the other way, you get this (and when you let go, the band will snap back and have nothing to do with the post):



            enter image description here






            share|cite|improve this answer









            $endgroup$

















              13












              $begingroup$

              As @Ben says, the key is that the middle point, where you switch from finishing the first loop to starting the second loop, is able to move. Here's some bad pictures to feed your intuition a bit.



              If you wrap a rubber band once around a post and let go, it will stay around the post:



              enter image description here



              On the other hand, if you wrap it once, then wrap it the other way, you get this (and when you let go, the band will snap back and have nothing to do with the post):



              enter image description here






              share|cite|improve this answer









              $endgroup$















                13












                13








                13





                $begingroup$

                As @Ben says, the key is that the middle point, where you switch from finishing the first loop to starting the second loop, is able to move. Here's some bad pictures to feed your intuition a bit.



                If you wrap a rubber band once around a post and let go, it will stay around the post:



                enter image description here



                On the other hand, if you wrap it once, then wrap it the other way, you get this (and when you let go, the band will snap back and have nothing to do with the post):



                enter image description here






                share|cite|improve this answer









                $endgroup$



                As @Ben says, the key is that the middle point, where you switch from finishing the first loop to starting the second loop, is able to move. Here's some bad pictures to feed your intuition a bit.



                If you wrap a rubber band once around a post and let go, it will stay around the post:



                enter image description here



                On the other hand, if you wrap it once, then wrap it the other way, you get this (and when you let go, the band will snap back and have nothing to do with the post):



                enter image description here







                share|cite|improve this answer












                share|cite|improve this answer



                share|cite|improve this answer










                answered Jan 30 at 16:06









                cspruncsprun

                1,40828




                1,40828





















                    9












                    $begingroup$

                    The reason one clockwise/counterclockwise loop is not nulhomotopic is that both the beginning and end of the loop must stay fixed. If you were allowed to move either one of the endpoints during the homotopy, then this path would be contractible. If you do one clockwise then one counter clockwise loop, then only the start of the first clockwise loop and end of the second counterclockwise loop must remain fixed, while the middle point is allowed to move. This allows you to contract the whole loop.



                    Concretely, if
                    $$f(t) = begincases e^4pi i t&tleq 1/2\ e^-4pi i t&1/2leq t leq 1 endcases$$



                    then there is a nulhomotopy $$h(t,s) = begincasesf(ts)&t leq 1/2\f(s(1-t)) & 1/2leq tendcases$$






                    share|cite|improve this answer











                    $endgroup$








                    • 1




                      $begingroup$
                      Presumably start and beginning should be beginning and end?
                      $endgroup$
                      – timtfj
                      Jan 30 at 14:39










                    • $begingroup$
                      @timtfj Fair enough :)
                      $endgroup$
                      – Ben
                      Jan 30 at 15:45










                    • $begingroup$
                      The $h$ you provide is not nullhomotopic. Indeed, check the endpoints are fixed for all values of $s$. they are correct for $0$ and $1$ but nowhere in between. What you describe is a line that makes it $frac1s$ of the way around the path that is one loop one way, one loop back. The problems are: you wish the condition ($t$ less/greater than $0.5$) to remain true of $t$, not the function argument, and, the function will not be smooth at switch in the current form.
                      $endgroup$
                      – drjpizzle
                      Jan 31 at 11:15











                    • $begingroup$
                      @drjpizzle Yes I can see it's not right - I think $f(st)$ should be correct for $t leq 1/2$, but not when $t geq 1/2$.
                      $endgroup$
                      – Ben
                      Jan 31 at 11:19










                    • $begingroup$
                      Well it's smooth and you have a nulhomotopy exists so you can extend it to one... I have put the formula as a comment to my answer (in response to yours.)
                      $endgroup$
                      – drjpizzle
                      Jan 31 at 11:36















                    9












                    $begingroup$

                    The reason one clockwise/counterclockwise loop is not nulhomotopic is that both the beginning and end of the loop must stay fixed. If you were allowed to move either one of the endpoints during the homotopy, then this path would be contractible. If you do one clockwise then one counter clockwise loop, then only the start of the first clockwise loop and end of the second counterclockwise loop must remain fixed, while the middle point is allowed to move. This allows you to contract the whole loop.



                    Concretely, if
                    $$f(t) = begincases e^4pi i t&tleq 1/2\ e^-4pi i t&1/2leq t leq 1 endcases$$



                    then there is a nulhomotopy $$h(t,s) = begincasesf(ts)&t leq 1/2\f(s(1-t)) & 1/2leq tendcases$$






                    share|cite|improve this answer











                    $endgroup$








                    • 1




                      $begingroup$
                      Presumably start and beginning should be beginning and end?
                      $endgroup$
                      – timtfj
                      Jan 30 at 14:39










                    • $begingroup$
                      @timtfj Fair enough :)
                      $endgroup$
                      – Ben
                      Jan 30 at 15:45










                    • $begingroup$
                      The $h$ you provide is not nullhomotopic. Indeed, check the endpoints are fixed for all values of $s$. they are correct for $0$ and $1$ but nowhere in between. What you describe is a line that makes it $frac1s$ of the way around the path that is one loop one way, one loop back. The problems are: you wish the condition ($t$ less/greater than $0.5$) to remain true of $t$, not the function argument, and, the function will not be smooth at switch in the current form.
                      $endgroup$
                      – drjpizzle
                      Jan 31 at 11:15











                    • $begingroup$
                      @drjpizzle Yes I can see it's not right - I think $f(st)$ should be correct for $t leq 1/2$, but not when $t geq 1/2$.
                      $endgroup$
                      – Ben
                      Jan 31 at 11:19










                    • $begingroup$
                      Well it's smooth and you have a nulhomotopy exists so you can extend it to one... I have put the formula as a comment to my answer (in response to yours.)
                      $endgroup$
                      – drjpizzle
                      Jan 31 at 11:36













                    9












                    9








                    9





                    $begingroup$

                    The reason one clockwise/counterclockwise loop is not nulhomotopic is that both the beginning and end of the loop must stay fixed. If you were allowed to move either one of the endpoints during the homotopy, then this path would be contractible. If you do one clockwise then one counter clockwise loop, then only the start of the first clockwise loop and end of the second counterclockwise loop must remain fixed, while the middle point is allowed to move. This allows you to contract the whole loop.



                    Concretely, if
                    $$f(t) = begincases e^4pi i t&tleq 1/2\ e^-4pi i t&1/2leq t leq 1 endcases$$



                    then there is a nulhomotopy $$h(t,s) = begincasesf(ts)&t leq 1/2\f(s(1-t)) & 1/2leq tendcases$$






                    share|cite|improve this answer











                    $endgroup$



                    The reason one clockwise/counterclockwise loop is not nulhomotopic is that both the beginning and end of the loop must stay fixed. If you were allowed to move either one of the endpoints during the homotopy, then this path would be contractible. If you do one clockwise then one counter clockwise loop, then only the start of the first clockwise loop and end of the second counterclockwise loop must remain fixed, while the middle point is allowed to move. This allows you to contract the whole loop.



                    Concretely, if
                    $$f(t) = begincases e^4pi i t&tleq 1/2\ e^-4pi i t&1/2leq t leq 1 endcases$$



                    then there is a nulhomotopy $$h(t,s) = begincasesf(ts)&t leq 1/2\f(s(1-t)) & 1/2leq tendcases$$







                    share|cite|improve this answer














                    share|cite|improve this answer



                    share|cite|improve this answer








                    edited Jan 31 at 11:36

























                    answered Jan 30 at 13:25









                    BenBen

                    4,011617




                    4,011617







                    • 1




                      $begingroup$
                      Presumably start and beginning should be beginning and end?
                      $endgroup$
                      – timtfj
                      Jan 30 at 14:39










                    • $begingroup$
                      @timtfj Fair enough :)
                      $endgroup$
                      – Ben
                      Jan 30 at 15:45










                    • $begingroup$
                      The $h$ you provide is not nullhomotopic. Indeed, check the endpoints are fixed for all values of $s$. they are correct for $0$ and $1$ but nowhere in between. What you describe is a line that makes it $frac1s$ of the way around the path that is one loop one way, one loop back. The problems are: you wish the condition ($t$ less/greater than $0.5$) to remain true of $t$, not the function argument, and, the function will not be smooth at switch in the current form.
                      $endgroup$
                      – drjpizzle
                      Jan 31 at 11:15











                    • $begingroup$
                      @drjpizzle Yes I can see it's not right - I think $f(st)$ should be correct for $t leq 1/2$, but not when $t geq 1/2$.
                      $endgroup$
                      – Ben
                      Jan 31 at 11:19










                    • $begingroup$
                      Well it's smooth and you have a nulhomotopy exists so you can extend it to one... I have put the formula as a comment to my answer (in response to yours.)
                      $endgroup$
                      – drjpizzle
                      Jan 31 at 11:36












                    • 1




                      $begingroup$
                      Presumably start and beginning should be beginning and end?
                      $endgroup$
                      – timtfj
                      Jan 30 at 14:39










                    • $begingroup$
                      @timtfj Fair enough :)
                      $endgroup$
                      – Ben
                      Jan 30 at 15:45










                    • $begingroup$
                      The $h$ you provide is not nullhomotopic. Indeed, check the endpoints are fixed for all values of $s$. they are correct for $0$ and $1$ but nowhere in between. What you describe is a line that makes it $frac1s$ of the way around the path that is one loop one way, one loop back. The problems are: you wish the condition ($t$ less/greater than $0.5$) to remain true of $t$, not the function argument, and, the function will not be smooth at switch in the current form.
                      $endgroup$
                      – drjpizzle
                      Jan 31 at 11:15











                    • $begingroup$
                      @drjpizzle Yes I can see it's not right - I think $f(st)$ should be correct for $t leq 1/2$, but not when $t geq 1/2$.
                      $endgroup$
                      – Ben
                      Jan 31 at 11:19










                    • $begingroup$
                      Well it's smooth and you have a nulhomotopy exists so you can extend it to one... I have put the formula as a comment to my answer (in response to yours.)
                      $endgroup$
                      – drjpizzle
                      Jan 31 at 11:36







                    1




                    1




                    $begingroup$
                    Presumably start and beginning should be beginning and end?
                    $endgroup$
                    – timtfj
                    Jan 30 at 14:39




                    $begingroup$
                    Presumably start and beginning should be beginning and end?
                    $endgroup$
                    – timtfj
                    Jan 30 at 14:39












                    $begingroup$
                    @timtfj Fair enough :)
                    $endgroup$
                    – Ben
                    Jan 30 at 15:45




                    $begingroup$
                    @timtfj Fair enough :)
                    $endgroup$
                    – Ben
                    Jan 30 at 15:45












                    $begingroup$
                    The $h$ you provide is not nullhomotopic. Indeed, check the endpoints are fixed for all values of $s$. they are correct for $0$ and $1$ but nowhere in between. What you describe is a line that makes it $frac1s$ of the way around the path that is one loop one way, one loop back. The problems are: you wish the condition ($t$ less/greater than $0.5$) to remain true of $t$, not the function argument, and, the function will not be smooth at switch in the current form.
                    $endgroup$
                    – drjpizzle
                    Jan 31 at 11:15





                    $begingroup$
                    The $h$ you provide is not nullhomotopic. Indeed, check the endpoints are fixed for all values of $s$. they are correct for $0$ and $1$ but nowhere in between. What you describe is a line that makes it $frac1s$ of the way around the path that is one loop one way, one loop back. The problems are: you wish the condition ($t$ less/greater than $0.5$) to remain true of $t$, not the function argument, and, the function will not be smooth at switch in the current form.
                    $endgroup$
                    – drjpizzle
                    Jan 31 at 11:15













                    $begingroup$
                    @drjpizzle Yes I can see it's not right - I think $f(st)$ should be correct for $t leq 1/2$, but not when $t geq 1/2$.
                    $endgroup$
                    – Ben
                    Jan 31 at 11:19




                    $begingroup$
                    @drjpizzle Yes I can see it's not right - I think $f(st)$ should be correct for $t leq 1/2$, but not when $t geq 1/2$.
                    $endgroup$
                    – Ben
                    Jan 31 at 11:19












                    $begingroup$
                    Well it's smooth and you have a nulhomotopy exists so you can extend it to one... I have put the formula as a comment to my answer (in response to yours.)
                    $endgroup$
                    – drjpizzle
                    Jan 31 at 11:36




                    $begingroup$
                    Well it's smooth and you have a nulhomotopy exists so you can extend it to one... I have put the formula as a comment to my answer (in response to yours.)
                    $endgroup$
                    – drjpizzle
                    Jan 31 at 11:36











                    0












                    $begingroup$

                    I think this is best answered directly/constructively:



                    Going a fraction $k$ about the circle one way and them coming back again, as $k$ varies smoothly between $0$ and $1$ should do the trick.



                    At $k = 0$ this is the point. At $k = 1$ this is one around and once back. I think its clear it's continuious.






                    share|cite|improve this answer









                    $endgroup$












                    • $begingroup$
                      Why it is right in the general case (the winding number arguments given) is a useful result and nice answers. It's the right way to think about it, and understand why this is a nice result. But to the particular question answered, I think this is the most direct answer.
                      $endgroup$
                      – drjpizzle
                      Jan 30 at 20:30











                    • $begingroup$
                      By the way this is what the nulhomtopy $h(t,s)$ in a previous answer does, if you plug in $k$ (in this answer) in the $s$ variable.
                      $endgroup$
                      – Ben
                      Jan 31 at 10:44










                    • $begingroup$
                      @Ben This is not the case. In your example, take $f(st)$ for $s = 0.5$. This becomes: $e^2 pi i t$ or one full loop. Not half a loop forward and half a loop back. I know what you are getting at but I think you made a mistake. I will comment on your answer to see if i can make this clear.
                      $endgroup$
                      – drjpizzle
                      Jan 31 at 11:06











                    • $begingroup$
                      Thanks you're right. I'll try to fix the formula for $h$, if you have any suggestions please let me know.
                      $endgroup$
                      – Ben
                      Jan 31 at 11:14










                    • $begingroup$
                      @Ben I think in this case, a wordy answer might be best. But: $h(t,s) = e^2s ((2 pi) t i)$ when $t < 0.5$, $e^2s ((2 pi) (1 - t) i)$ otherwise, I think is correct. edit: grammar and silly mistakes.
                      $endgroup$
                      – drjpizzle
                      Jan 31 at 11:25
















                    0












                    $begingroup$

                    I think this is best answered directly/constructively:



                    Going a fraction $k$ about the circle one way and them coming back again, as $k$ varies smoothly between $0$ and $1$ should do the trick.



                    At $k = 0$ this is the point. At $k = 1$ this is one around and once back. I think its clear it's continuious.






                    share|cite|improve this answer









                    $endgroup$












                    • $begingroup$
                      Why it is right in the general case (the winding number arguments given) is a useful result and nice answers. It's the right way to think about it, and understand why this is a nice result. But to the particular question answered, I think this is the most direct answer.
                      $endgroup$
                      – drjpizzle
                      Jan 30 at 20:30











                    • $begingroup$
                      By the way this is what the nulhomtopy $h(t,s)$ in a previous answer does, if you plug in $k$ (in this answer) in the $s$ variable.
                      $endgroup$
                      – Ben
                      Jan 31 at 10:44










                    • $begingroup$
                      @Ben This is not the case. In your example, take $f(st)$ for $s = 0.5$. This becomes: $e^2 pi i t$ or one full loop. Not half a loop forward and half a loop back. I know what you are getting at but I think you made a mistake. I will comment on your answer to see if i can make this clear.
                      $endgroup$
                      – drjpizzle
                      Jan 31 at 11:06











                    • $begingroup$
                      Thanks you're right. I'll try to fix the formula for $h$, if you have any suggestions please let me know.
                      $endgroup$
                      – Ben
                      Jan 31 at 11:14










                    • $begingroup$
                      @Ben I think in this case, a wordy answer might be best. But: $h(t,s) = e^2s ((2 pi) t i)$ when $t < 0.5$, $e^2s ((2 pi) (1 - t) i)$ otherwise, I think is correct. edit: grammar and silly mistakes.
                      $endgroup$
                      – drjpizzle
                      Jan 31 at 11:25














                    0












                    0








                    0





                    $begingroup$

                    I think this is best answered directly/constructively:



                    Going a fraction $k$ about the circle one way and them coming back again, as $k$ varies smoothly between $0$ and $1$ should do the trick.



                    At $k = 0$ this is the point. At $k = 1$ this is one around and once back. I think its clear it's continuious.






                    share|cite|improve this answer









                    $endgroup$



                    I think this is best answered directly/constructively:



                    Going a fraction $k$ about the circle one way and them coming back again, as $k$ varies smoothly between $0$ and $1$ should do the trick.



                    At $k = 0$ this is the point. At $k = 1$ this is one around and once back. I think its clear it's continuious.







                    share|cite|improve this answer












                    share|cite|improve this answer



                    share|cite|improve this answer










                    answered Jan 30 at 20:28









                    drjpizzledrjpizzle

                    1712




                    1712











                    • $begingroup$
                      Why it is right in the general case (the winding number arguments given) is a useful result and nice answers. It's the right way to think about it, and understand why this is a nice result. But to the particular question answered, I think this is the most direct answer.
                      $endgroup$
                      – drjpizzle
                      Jan 30 at 20:30











                    • $begingroup$
                      By the way this is what the nulhomtopy $h(t,s)$ in a previous answer does, if you plug in $k$ (in this answer) in the $s$ variable.
                      $endgroup$
                      – Ben
                      Jan 31 at 10:44










                    • $begingroup$
                      @Ben This is not the case. In your example, take $f(st)$ for $s = 0.5$. This becomes: $e^2 pi i t$ or one full loop. Not half a loop forward and half a loop back. I know what you are getting at but I think you made a mistake. I will comment on your answer to see if i can make this clear.
                      $endgroup$
                      – drjpizzle
                      Jan 31 at 11:06











                    • $begingroup$
                      Thanks you're right. I'll try to fix the formula for $h$, if you have any suggestions please let me know.
                      $endgroup$
                      – Ben
                      Jan 31 at 11:14










                    • $begingroup$
                      @Ben I think in this case, a wordy answer might be best. But: $h(t,s) = e^2s ((2 pi) t i)$ when $t < 0.5$, $e^2s ((2 pi) (1 - t) i)$ otherwise, I think is correct. edit: grammar and silly mistakes.
                      $endgroup$
                      – drjpizzle
                      Jan 31 at 11:25

















                    • $begingroup$
                      Why it is right in the general case (the winding number arguments given) is a useful result and nice answers. It's the right way to think about it, and understand why this is a nice result. But to the particular question answered, I think this is the most direct answer.
                      $endgroup$
                      – drjpizzle
                      Jan 30 at 20:30











                    • $begingroup$
                      By the way this is what the nulhomtopy $h(t,s)$ in a previous answer does, if you plug in $k$ (in this answer) in the $s$ variable.
                      $endgroup$
                      – Ben
                      Jan 31 at 10:44










                    • $begingroup$
                      @Ben This is not the case. In your example, take $f(st)$ for $s = 0.5$. This becomes: $e^2 pi i t$ or one full loop. Not half a loop forward and half a loop back. I know what you are getting at but I think you made a mistake. I will comment on your answer to see if i can make this clear.
                      $endgroup$
                      – drjpizzle
                      Jan 31 at 11:06











                    • $begingroup$
                      Thanks you're right. I'll try to fix the formula for $h$, if you have any suggestions please let me know.
                      $endgroup$
                      – Ben
                      Jan 31 at 11:14










                    • $begingroup$
                      @Ben I think in this case, a wordy answer might be best. But: $h(t,s) = e^2s ((2 pi) t i)$ when $t < 0.5$, $e^2s ((2 pi) (1 - t) i)$ otherwise, I think is correct. edit: grammar and silly mistakes.
                      $endgroup$
                      – drjpizzle
                      Jan 31 at 11:25
















                    $begingroup$
                    Why it is right in the general case (the winding number arguments given) is a useful result and nice answers. It's the right way to think about it, and understand why this is a nice result. But to the particular question answered, I think this is the most direct answer.
                    $endgroup$
                    – drjpizzle
                    Jan 30 at 20:30





                    $begingroup$
                    Why it is right in the general case (the winding number arguments given) is a useful result and nice answers. It's the right way to think about it, and understand why this is a nice result. But to the particular question answered, I think this is the most direct answer.
                    $endgroup$
                    – drjpizzle
                    Jan 30 at 20:30













                    $begingroup$
                    By the way this is what the nulhomtopy $h(t,s)$ in a previous answer does, if you plug in $k$ (in this answer) in the $s$ variable.
                    $endgroup$
                    – Ben
                    Jan 31 at 10:44




                    $begingroup$
                    By the way this is what the nulhomtopy $h(t,s)$ in a previous answer does, if you plug in $k$ (in this answer) in the $s$ variable.
                    $endgroup$
                    – Ben
                    Jan 31 at 10:44












                    $begingroup$
                    @Ben This is not the case. In your example, take $f(st)$ for $s = 0.5$. This becomes: $e^2 pi i t$ or one full loop. Not half a loop forward and half a loop back. I know what you are getting at but I think you made a mistake. I will comment on your answer to see if i can make this clear.
                    $endgroup$
                    – drjpizzle
                    Jan 31 at 11:06





                    $begingroup$
                    @Ben This is not the case. In your example, take $f(st)$ for $s = 0.5$. This becomes: $e^2 pi i t$ or one full loop. Not half a loop forward and half a loop back. I know what you are getting at but I think you made a mistake. I will comment on your answer to see if i can make this clear.
                    $endgroup$
                    – drjpizzle
                    Jan 31 at 11:06













                    $begingroup$
                    Thanks you're right. I'll try to fix the formula for $h$, if you have any suggestions please let me know.
                    $endgroup$
                    – Ben
                    Jan 31 at 11:14




                    $begingroup$
                    Thanks you're right. I'll try to fix the formula for $h$, if you have any suggestions please let me know.
                    $endgroup$
                    – Ben
                    Jan 31 at 11:14












                    $begingroup$
                    @Ben I think in this case, a wordy answer might be best. But: $h(t,s) = e^2s ((2 pi) t i)$ when $t < 0.5$, $e^2s ((2 pi) (1 - t) i)$ otherwise, I think is correct. edit: grammar and silly mistakes.
                    $endgroup$
                    – drjpizzle
                    Jan 31 at 11:25





                    $begingroup$
                    @Ben I think in this case, a wordy answer might be best. But: $h(t,s) = e^2s ((2 pi) t i)$ when $t < 0.5$, $e^2s ((2 pi) (1 - t) i)$ otherwise, I think is correct. edit: grammar and silly mistakes.
                    $endgroup$
                    – drjpizzle
                    Jan 31 at 11:25


















                    draft saved

                    draft discarded
















































                    Thanks for contributing an answer to Mathematics Stack Exchange!


                    • Please be sure to answer the question. Provide details and share your research!

                    But avoid


                    • Asking for help, clarification, or responding to other answers.

                    • Making statements based on opinion; back them up with references or personal experience.

                    Use MathJax to format equations. MathJax reference.


                    To learn more, see our tips on writing great answers.




                    draft saved


                    draft discarded














                    StackExchange.ready(
                    function ()
                    StackExchange.openid.initPostLogin('.new-post-login', 'https%3a%2f%2fmath.stackexchange.com%2fquestions%2f3093470%2fhow-is-going-around-the-circle-once-in-each-direction-homotopic-to-a-point%23new-answer', 'question_page');

                    );

                    Post as a guest















                    Required, but never shown





















































                    Required, but never shown














                    Required, but never shown












                    Required, but never shown







                    Required, but never shown

































                    Required, but never shown














                    Required, but never shown












                    Required, but never shown







                    Required, but never shown






                    Popular posts from this blog

                    How to check contact read email or not when send email to Individual?

                    Bahrain

                    Postfix configuration issue with fips on centos 7; mailgun relay